Quantcast
  • Register
PhysicsOverflow is a next-generation academic platform for physicists and astronomers, including a community peer review system and a postgraduate-level discussion forum analogous to MathOverflow.

Welcome to PhysicsOverflow! PhysicsOverflow is an open platform for community peer review and graduate-level Physics discussion.

Please help promote PhysicsOverflow ads elsewhere if you like it.

News

PO is now at the Physics Department of Bielefeld University!

New printer friendly PO pages!

Migration to Bielefeld University was successful!

Please vote for this year's PhysicsOverflow ads!

Please do help out in categorising submissions. Submit a paper to PhysicsOverflow!

... see more

Tools for paper authors

Submit paper
Claim Paper Authorship

Tools for SE users

Search User
Reclaim SE Account
Request Account Merger
Nativise imported posts
Claim post (deleted users)
Import SE post

Users whose questions have been imported from Physics Stack Exchange, Theoretical Physics Stack Exchange, or any other Stack Exchange site are kindly requested to reclaim their account and not to register as a new user.

Public \(\beta\) tools

Report a bug with a feature
Request a new functionality
404 page design
Send feedback

Attributions

(propose a free ad)

Site Statistics

205 submissions , 163 unreviewed
5,047 questions , 2,200 unanswered
5,345 answers , 22,709 comments
1,470 users with positive rep
816 active unimported users
More ...

  Double connectivity of $SO(3)$ group manifold

+ 6 like - 0 dislike
2821 views

Is there any physical significance of the fact that the group manifold (parameter space) of $SO(3)$ is doubly connected?

EDIT 1: Let me clarify my question. It was too vague. There exists two equivalence classes of paths in the group manifold of SO(3) or in other words, $\Pi_1(SO(3))=Z_2$. This space is therefore doubly connected. There are paths which come back to initial configurations after a rotation of $2\pi$ and others after a rotation of $4\pi$, with proper parametrization of angles.

Using this fact, is it possible to show that such a topology admits the existence of half-integer spins and integer spins? I understand spinors as objects whose wavefunctions pick up a -ve sign after a rotation of $2\pi$, and comes back to itself after a rotation of $4\pi$. Right? But from the topological argument given above, it is not clear to me, that how does it lead to two kinds of wavefunctions, spinor-type $(j=\frac{1}{2},\frac{3}{2},\frac{5}{2}...)$ and tensor-type $j=0,1,2,...$? It is not explicitly clear how these two types of paths in SO(3) group manifold will lead to such transformation properties on "the wavefunctions"?

EDIT 2: http://en.wikipedia.org/wiki/SO%283%29#Topology.

This post imported from StackExchange Physics at 2014-04-12 19:04 (UCT), posted by SE-user Roopam
asked Feb 3, 2014 in Theoretical Physics by Roopam (145 points) [ no revision ]
Most voted comments show all comments
Dear @RoopamSinha: May I suggest you should delete your last sentence and ask it as an excellent separate question. It kind of intuitively implies that anyon spins can be any rational number - you can have an irrep where it takes $N$ full turn to get a $2\pi$ phase, so that each full turn adds $2\pi/N$ phase. Thence one can see that any phase of the form $m/n$ is possible.

This post imported from StackExchange Physics at 2014-04-12 19:04 (UCT), posted by SE-user WetSavannaAnimal aka Rod Vance
@WetSavannaAnimalakaRodVance Well, to be honest, I still don't feel I understand what he intends to ask. The title of the question is also misleading, at least doesn't match the "EDIT" (the post before the edit contained only one line). Also, if he does intend to ask for a topological perspective on spin-statistics theorem, I think I can provide an answer or some useful reference but this should have been clearly stated in the post. This is not just for the sake of OP but also fellow users of this website.

This post imported from StackExchange Physics at 2014-04-12 19:04 (UCT), posted by SE-user Isidore Seville
@IsidoreSeville I agree it would be wise to wait until OP has read the other answers in the links given and then refines his question so you don't waste your work. However, if, as you say, there is a topological perspective on spin-statistics then I'd ask the question to see your answer!

This post imported from StackExchange Physics at 2014-04-12 19:04 (UCT), posted by SE-user WetSavannaAnimal aka Rod Vance
Dear WetSavannaAnimal aka Rod Vance- "...*have established that there are integer and half integer spins*..." I have not quite understood this fact too. So I re-edited my questions. How is this topological fact related to the two types of representations of SO(3), a class labeled by half-integer j values and another class labeled by integral j values? This is the first issue which I'm stuck at.

This post imported from StackExchange Physics at 2014-04-12 19:04 (UCT), posted by SE-user Roopam
physics.stackexchange.com/questions/147/…

This post imported from StackExchange Physics at 2014-04-12 19:04 (UCT), posted by SE-user Kostya
Most recent comments show all comments
Comment to the question (v1): You mean apart from the fact that the Lie group $SO(3)$ doesn't have even-dimensional (=half-integer spin) irreps?

This post imported from StackExchange Physics at 2014-04-12 19:04 (UCT), posted by SE-user Qmechanic
Related question by OP: physics.stackexchange.com/q/96542/2451 Related: physics.stackexchange.com/q/13787/2451 , physics.stackexchange.com/q/65767/2451 and links therein.

This post imported from StackExchange Physics at 2014-04-12 19:04 (UCT), posted by SE-user Qmechanic

1 Answer

+ 6 like - 0 dislike

Just in view of the double universal covering provided by $SU(2)$, $SO(3)$ must a quotient of $SU(2)$ with respect to a central discrete normal subgroup with two elements. This is consequence of a general property of universal covering Lie groups:

If $\pi: \tilde{G} \to G$ is the universal covering Lie-group homomorphism, the kernel $H$ of $\pi$ is a discrete normal central subgroup of the universal covering $\tilde{G}$ of $G= \tilde{G}/H$, and $H$ is isomorphic to the fundamental group of $G$, i.e. $\pi_1(G)$ (wich, for Lie groups, is Abelian) .

One element of that subgroup must be $I$ (since a group includes the neutral element). The other, $J$, must verify $JJ=I$ and thus $J=J^{-1}= J^\dagger$. By direct inspection one sees that in $SU(2)$ it is only possible for $J= -I$. So $SO(3) = SU(2)/\{I,-I\}$.

Notice that $\{I,-I\} = \{e^{i4\pi \vec{n}\cdot \vec{\sigma}/2 }, e^{i2\pi \vec{n}\cdot \vec{\sigma}/2 }\}$ stays in the center of $SU(2)$, namely the elements of this subgroup commute with all of the elements of $SU(2)$. Moreover $\{I,-I\}=: \mathbb Z_2$ is just the first homotopy group of $SO(3)$ as it must be in view of the general statement I quoted above.

A unitary representations of $SO(3)$ is also a representation of $SU(2)$ through the projection Lie group homomorphism $\pi: SU(2) \to SU(2)/\{I,-I\} = SO(3)$. So, studying unitary reps of $SU(2)$ covers the whole class of unitary reps of $SO(3)$. Let us study those reps.

Consider a unitary representation $U$ of $SU(2)$ in the Hilbert space $H$. The central subgroup $\{I,-I\}$ must be represented by $U(I)= I_H$ and $U(-I)= J_H$, but $J_HJ_H= I_H$ so, as before, $J_H= J_H^{-1}= J_H^\dagger$.

As $J_H$ is unitary and self-adjoint simultaneously, its spectrum has to be included in $\mathbb R \cap \{\lambda \in \mathbb C \:|\: |\lambda|=1\}$. So (a) it is made of $\pm 1$ at most and (b) the spectrum is a pure point spectrum and so only proper eigenspeces arise in its spectral decomposition.

If $-1$ is not present in the spectrum, the only eigenvalue is $1$ and thus $U(-I)= I_H$. If only the eigenvalue $-1$ is present, instead, $U(-I)= -I_H$.

If the representation is irreducible $\pm 1$ cannot be simultaneously eigenvalues. Otherwise $H$ would be split into the orthogonal direct sum of eigenspaces $H_{+1}\oplus H_{-1}$. As $U(-1)=J_H$ commutes with all $U(g)$ (because $-I$ is in the center of $SU(2)$ and $U$ is a representation), $H_{+1}$ and $H_{-1}$ would be invariant subspaces for all the representation and it is forbidden as $U$ is irreducible.

We conclude that,

if $U$ is an irreducible unitary representation of $SU(2)$, the discrete normal subgroup $\{I,-I\}$ can only be represented by either $\{I_H\}$ or $\{I_H, -I_H\}$.

Moreover:

Since $SO(3) = SU(2)/\{I,-I\}$, in the former case $U$ is also a representation of $SO(3)$. It means that $I = e^{i 4\pi \vec{n}\cdot \vec{\sigma} }$ and $e^{i 2\pi \vec{n}\cdot \vec{\sigma}/2 } = -I$ are both transformed into $I_H$ by $U$.

In the latter case, instead, $U$ is not a true representation of $SO(3)$, just in view of a sign appearing after $2\pi$, because $e^{i 2\pi \vec{n}\cdot \vec{\sigma}/2 } = -I$ is transformed into $-I_H$ and only $I = e^{i 4\pi \vec{n}\cdot \vec{\sigma}/2 }$ is transformed into $I$ by $U$.

This post imported from StackExchange Physics at 2014-04-12 19:04 (UCT), posted by SE-user V. Moretti
answered Feb 5, 2014 by Valter Moretti (2,085 points) [ no revision ]
@V.Moretti- Amazing. This answer is very insightful. But I still wonder where does the topology of the SO(3) group manifold (which I mentioned in the question) enter this business? It appears to me that the above answer uses the fact that there is a 2-to-1 homomorphism between SU(2) and SO(3). Or am I missing the connection?

This post imported from StackExchange Physics at 2014-04-12 19:04 (UCT), posted by SE-user Roopam
SO(3) is a Lie group with fundamental homotopy group $\mathbb Z_2$. Therefore its universal covering group is a simply connected Lie group and $SO(2)$ is obtained by taking the quotient of the latter and a discrete normal subgroup with two elements. That two is the "topological" information: It remembers the structure of the first homotopy group of the manifold $SO(3)$. Knowing that the universal covering is $SU(2)$ one has to look for a discrete subgroup containing only two elements...

This post imported from StackExchange Physics at 2014-04-12 19:04 (UCT), posted by SE-user V. Moretti
@V.Moretti- Okay, I understand. Now it is absolutely clear. Thanks.

This post imported from StackExchange Physics at 2014-04-12 19:04 (UCT), posted by SE-user Roopam
I made precise the relevant topological relation in the text just expanding a comment.

This post imported from StackExchange Physics at 2014-04-12 19:04 (UCT), posted by SE-user V. Moretti
@V.Moretti I'm sure you meant to say this in your comment above but in general you have to look for a discrete normal subgroup containing only two elements. These by Schreier's theorem must then be subgroups of the centre. Only being pedantic because the OP's clearly very interested in the details, I wouldn't have mentioned it otherwise.

This post imported from StackExchange Physics at 2014-04-12 19:04 (UCT), posted by SE-user WetSavannaAnimal aka Rod Vance
@RoopamSinha Comment to V.Moretti was meant for you too. And I must apologise for not answering your question to me under your original post - somehow I must have gotten a few pings at once and I lost track of them all. Unfortunately, the little red light only goes on once - maybe it should stay lit until you've clicked on all your unread pings.

This post imported from StackExchange Physics at 2014-04-12 19:04 (UCT), posted by SE-user WetSavannaAnimal aka Rod Vance
@RoopamSinha The theorem I cited is not what Schreier is most famous for - I'm looking for an online reference for you now.

This post imported from StackExchange Physics at 2014-04-12 19:04 (UCT), posted by SE-user WetSavannaAnimal aka Rod Vance
@WetSavannaAnimal aka Rod Vance Thank you for your comment. Yes, I wrote in a hurry and I wrote something quite confused "discrete normal subgroup with two elements" (<<with two elements>> does not make much sense: Instead you wrote correctly: "a discrete normal subgroup containing only two elements". Thanks.

This post imported from StackExchange Physics at 2014-04-12 19:04 (UCT), posted by SE-user V. Moretti
@Roopam Look at top of page 70 of this (the theorem is stated at the bottom of p69, and you should be able to see that the method of this little gem of a proof works for any connected Lie group even though Stillwell cites "path connected matrix Lie group") ohkawa.cc.it-hiroshima.ac.jp/AoPS.pdf/MathTextBook/…

This post imported from StackExchange Physics at 2014-04-12 19:04 (UCT), posted by SE-user WetSavannaAnimal aka Rod Vance

Your answer

Please use answers only to (at least partly) answer questions. To comment, discuss, or ask for clarification, leave a comment instead.
To mask links under text, please type your text, highlight it, and click the "link" button. You can then enter your link URL.
Please consult the FAQ for as to how to format your post.
This is the answer box; if you want to write a comment instead, please use the 'add comment' button.
Live preview (may slow down editor)   Preview
Your name to display (optional):
Privacy: Your email address will only be used for sending these notifications.
Anti-spam verification:
If you are a human please identify the position of the character covered by the symbol $\varnothing$ in the following word:
p$\hbar$ysicsO$\varnothing$erflow
Then drag the red bullet below over the corresponding character of our banner. When you drop it there, the bullet changes to green (on slow internet connections after a few seconds).
Please complete the anti-spam verification




user contributions licensed under cc by-sa 3.0 with attribution required

Your rights
...